Added topic tags to some S&J-4 problems + minor typos
[course.git] / latex / problems / Serway_and_Jewett_4 / problem28.57.tex
1 \begin{problem*}{28.57} % photoelectric effect
2 The following table shows data obtained in a photoelectric
3 experiment.  \Part{a} Using these data, make a graph similar to Active
4 Figure 28.9 that plots as a straight line.  From the graph,
5 determine \Part{b} an experimental value for Planck's constant (in
6 joule-seconds) and \Part{c} the work function (in electron volts) for
7 the surface.  (Two significant figures for each answer are
8 sufficient.)
9 \begin{center}
10 \begin{tabular}{r r}
11 Wavelength (nm) & Maximum Kinetic Energy of Photoelectrons (eV) \\
12 $588$ & $0.67$ \\
13 $505$ & $0.98$ \\
14 $445$ & $1.35$ \\
15 $399$ & $1.63$
16 \end{tabular}
17 \end{center}
18 \end{problem*} % problem 28.57
19
20 \begin{solution}
21 \Part{a}
22 We can convert the wavelengths to frequencies and graph them
23 \begin{center}
24 \begin{tabular}{r r}
25 $\lambda$ (nm) & $f=c/\lambda$ (THz) \\
26 $588$ & $510$ \\
27 $505$ & $594$ \\
28 $445$ & $674$ \\
29 $399$ & $751$
30 \end{tabular} \\
31 \begin{asy}
32 import graph;
33 import stats;
34 size(8cm, 5cm, IgnoreAspect);
35
36 real c = 299792458.0;
37
38 real fmin = 0;
39 real fmax = 800;
40
41 real[] x = {588,  505,  445,  399};  // nm
42 real[] y = {0.67, 0.98, 1.35, 1.63}; // eV
43
44 // convert wavelength in nm to freq in THz
45 int i;
46 for (i=0; i<x.length; ++i) {
47   x[i] = c/x[i]/1e3;
48 }
49
50 linefit L=leastsquares(x, y);
51 write("work function");
52 write(L.b);
53 write("h/e");
54 write(L.m);
55
56 real fit(real f)
57 {
58   return L.b + L.m*f;
59 }
60
61 draw(graph(fit, fmin, fmax), red);
62 draw(graph(x, y), p=invisible, marker=marker(scale(1pt)*unitcircle, blue));
63
64 pen thin=linewidth(0.5*linewidth())+grey;
65 xaxis("$f$ (THz)",BottomTop,
66       LeftTicks(begin=false,end=false,extend=true,ptick=thin));
67 yaxis("$K_{max}$ (V)",LeftRight,
68       RightTicks(begin=false,end=false,extend=true,ptick=thin));
69 \end{asy}
70 \end{center}
71
72 The least-squares fit yields parameters \Part{c}
73 $\phi=\ans{1.4\U{eV}}$ and $h/e = 4.04\U{mV/THz}$.  So \Part{b}
74 $h=\ans{6.5\E{-34}\U{J$\cdot$s}}$.
75 \end{solution}